9000145403

Část: 
Project ID: 
9000145403
Accepted: 
1
Clonable: 
0
Easy: 
0
Je dána funkce \(f\colon y = \frac{4-3x} {x\left (1-x\right )}\). Vyberte pravdivé tvrzení:
V bodě \(x = \frac{2} {3}\) má funkce \(f\) lokální minimum.
V bodě \(x = \frac{2} {3}\) má funkce \(f\) lokální maximum.
Daná funkce \(f\) má na množině \(\mathbb{R}\setminus \{0{,}1\}\) globální maximum v bodě \(x = \frac{2} {3}\).
Daná funkce \(f\) má na množině \(\mathbb{R}\setminus \{0{,}1\}\) globální minimum v bodě \(x = \frac{2} {3}\).